LSAT and Law School Admissions Forum

Get expert LSAT preparation and law school admissions advice from PowerScore Test Preparation.

User avatar
 Dave Killoran
PowerScore Staff
  • PowerScore Staff
  • Posts: 5853
  • Joined: Mar 25, 2011
|
#88161
Complete Question Explanation
(The complete setup for this game can be found here: lsat/viewtopic.php?f=169&p=88159#p88159)

The correct answer choice is (C).

The condition in the question stem indicates that more women are on stage than men. Thus, at least one, two, or three women must be on stage. However, from the third rule, if just one woman is on stage, then G must be on the stage. Therefore, there must be at least one man on the stage at all times, and there has to be more than one woman (in order to meet the condition in the question stem). But, as shown in the discussion of the first two rules, both J and L can never be on stage at the same time, and so all three women cannot concurrently be on stage. Therefore, exactly two women must be on stage, and exactly one man (G) must be on stage. The correct answer is therefore three, or answer choice (C).

Although we have established three as the correct answer, for educational purposes let us take a moment to analyze the problem in more detail. Of the two women, one of those two women must be J or L. However, if J is on stage, then F, a man, must also be on stage. This would equalize the number of women and men on stage (F and G), and so J cannot be on stage. Therefore, L and K must be the two women on stage, along with one man, G.

Get the most out of your LSAT Prep Plus subscription.

Analyze and track your performance with our Testing and Analytics Package.